Đến nội dung

phuc_90 nội dung

Có 79 mục bởi phuc_90 (Tìm giới hạn từ 29-03-2020)



Sắp theo                Sắp xếp  

#737461 CM $I + A^2$ khả nghịch và hãy tìm nghịch đảo của $I + A^2$

Đã gửi bởi phuc_90 on 27-02-2023 - 17:01 trong Đại số tuyến tính, Hình học giải tích

Cho $A$ là ma trận vuông cấp $n \ge 2$ thỏa mãn $A^3 = 0$.
(a.) Chứng minh : $I + A + A^2$ khả nghịch và hãy tìm nghịch đảo của $I + A + A^2$
(b.) Chứng minh : $I + A^2$ khả nghịch và hãy tìm nghịch đảo của $I + A^2$

 

a)  $(I+A+A^2)^{-1} = I-A$

 

b)  $(I+A^2)^{-1} = I-A^2$




#735153 Cho $x,y,z$ là các số không âm. Chứng minh rằng: $4(xy+yz+zx)...

Đã gửi bởi phuc_90 on 30-09-2022 - 13:40 trong Tài liệu, chuyên đề, phương pháp về Số học

Gợi ý, áp dụng BĐT Cauchy - Schwarz :  $(ab+cd)^2 \leq (a^2+c^2)(b^2+d^2)$




#731427 Chứng minh ma trận khả nghịch

Đã gửi bởi phuc_90 on 02-11-2021 - 13:10 trong Tài liệu và chuyên đề Đại số tuyến tính và Hình học giải tích

Post hết lời giải lên mới biết được em.



#731419 $(E+A)^n$

Đã gửi bởi phuc_90 on 01-11-2021 - 14:51 trong Đại số tuyến tính, Hình học giải tích

Bài này không giải được ở chỗ nào thế bạn ?



#731368 Các kiến thức cơ bản về Supremum và Infimum

Đã gửi bởi phuc_90 on 29-10-2021 - 16:25 trong Giải tích

Có bạn hỏi tôi về cách tìm SupremumInfimum của một tập $A\subset \mathbb{R}$ như thế nào ? mong bài viết nho nhỏ này sẽ giúp các bạn hiểu rõ hơn về Supremum và Infimum của một tập hợp $A\subset \mathbb{R}$, từ đó có một phương pháp giải các bài toán dạng này cho riêng mình.

 

Định nghĩa

 

Cho $A$ là tập con khác rỗng của $\mathbb{R}$.

 

$\bullet$   Tập $A$ được gọi là bị chặn trên nếu $\exists M\in\mathbb{R}:\,\,\,a\leq M\,\,,\,\forall a\in A$.

 

Với $A$ là tập bị chặn trên thì Supremum của $A$, ký hiệu $Sup A$ là chặn trên nhỏ nhất của $A$, tức là nếu $m$ là một chặn trên của $A$ thì ta luôn có $Sup A\leq m$.  Nếu tập $A$ không bị chặn trên thì người ta đặt $Sup \,A=+\infty$.

 

$\bullet$   Tập $A$ được gọi là bị chặn dưới nếu $\exists M\in\mathbb{R}:\,\,\,a\geq M\,\,,\,\forall a\in A$.

 

Với $A$ là tập bị chặn dưới thì Infimum của $A$, ký hiệu $Inf A$ là chặn dưới lớn nhất của $A$, tức là nếu $n$ là một chặn dưới của $A$ thì ta luôn có $Inf A\geq n$.  Nếu tập $A$ không bị chặn dưới thì người ta đặt $Inf \,A=-\infty$

 

Một số kết quả liên quan đến Supremum và Infimum

 

Định lý 1:   Cho $A$ là tập con khác rỗng của $\mathbb{R}$ và bị chặn trên. $Sup A=m$ khi và chỉ khi $\left\{\begin{matrix}a\leq m\,\,,\,\forall a\in A\\ \forall \varepsilon >0\,\,,\,\exists a^*\in A:\,\,a^*>m-\varepsilon \end{matrix}\right.$.

 

Hơn nữa, nếu $m$ là một chặn trên của $A$ và $m\in A$ thì $Sup A=m$, lúc này $Sup A$ chính là Maximum của tập $A$.

 

Chứng minh

 

$\left ( \Rightarrow  \right )$  Giả sử $Sup A=m$

 

Nếu $\exists \varepsilon >0$ sao cho $m-\varepsilon$ là một chặn trên của $A$ thì $m-\varepsilon \geq Sup A=m$  (vô lý)

 

Vậy $\forall \varepsilon >0$ thì $m-\varepsilon$ không thể là một chặn trên của $A$ hay $\forall \varepsilon >0\,\,,\,\exists\,a^*\in A:\,\,\,a^*>m-\varepsilon$.

 

$\left ( \Leftarrow   \right )$   Giả sử   $\left\{\begin{matrix}a\leq m\,\,,\,\forall a\in A\\ \forall \varepsilon >0\,\,,\,\exists a^*\in A:\,\,a^*>m-\varepsilon \end{matrix}\right.$

 

Ta có $a\leq m\,\,,\,\forall a\in A$ nên $m$ là một chặn trên của $A$, do đó $Sup A\leq m$.

 

Đặt $d=m-Sup A\geq 0$, nếu $d>0$ thì theo giả thiết, tồn tại $a^*\in A:\,\,\,a^*>m-d=Sup A$  (vô lý).

 

Vậy $d=0$  hay  $Sup A=m$.

 

Định lý 2:   Cho $A$ là tập con khác rỗng của $\mathbb{R}$ và bị chặn dưới. $Inf A=n$ khi và chỉ khi $\left\{\begin{matrix}a\geq n\,\,,\,\forall a\in A\\ \forall \varepsilon >0\,\,,\,\exists a^*\in A:\,\,a^*<n+\varepsilon \end{matrix}\right.$.

 

Hơn nữa, nếu $n$ là một chặn dưới của $A$ và $n\in A$ thì $Inf A=n$, lúc này $Inf A$ chính là Minximum của tập $A$.

(Chứng minh xem như bài tập)

 

Định lý 3:   Nếu $(u_n)_n$ là dãy số thực và là dãy tăng thì $\lim_{n \to \infty } u_n=Sup\left \{ u_n\,|\,n\in \mathbb{N} \right \}$

 

Chứng minh

 

Nếu $(u_n)_n$ không bị chặn trên, tức là $\forall M>0\,,\,\exists n_0\in \mathbb{N}:\,\,u_{n_0}>M$  (*)  và  $Sup\left \{ u_n\,|\,n\in \mathbb{N} \right \}=+\infty$

 

Do $(u_n)_n$ là dãy đơn điệu tăng nên (*) được viết lại thành  $\forall M>0\,,\,\exists n_0\in \mathbb{N}\,,\,\forall n\geq n_0:\,\,u_n>M$

 

Đây chính là định nghĩa của $\lim_{n \to \infty }u_n=+\infty$. Suy ra $\lim_{n \to \infty } u_n=Sup\left \{ u_n\,|\,n\in \mathbb{N} \right \}$.

 

Nếu $(u_n)_n$ bị chặn trên, tức là $Sup\left \{ u_n\,|\,n\in \mathbb{N} \right \}$ tồn tại, đặt $Sup\left \{ u_n\,|\,n\in \mathbb{N} \right \}=a$

 

Khi đó, theo định lý 1 thì  $\forall \varepsilon >0\,,\,\exists n_0\in \mathbb{N}:\,\,a+\varepsilon >u_{n_0}>a-\varepsilon$, do $(u_n)_n$ là dãy đơn điệu tăng nên điều này được viết lại thành

 

$$\forall \varepsilon >0\,,\,\exists n_0\in \mathbb{N}\,,\,\forall n\geq n_0:\,\,\left | u_n-a \right |<\varepsilon$$

 

Suy ra $\lim_{n \to \infty }u_n=a=Sup\left \{ u_n\,|\,n\in \mathbb{N} \right \}$

 

Định lý 4:   Nếu $(u_n)_n$ là dãy số thực và là dãy giảm thì $\lim_{n \to \infty } u_n=Inf\left \{ u_n\,|\,n\in \mathbb{N} \right \}$

(Chứng minh xem như bài tập)

 

 

Phần áp dụng

 

Bài toán 1:   Cho  $A=\left \{ x\in \mathbb{Q}:\,\,x>0\,,\,x^2<2 \right \}$.  Chứng minh rằng  $Sup\,A=\sqrt{2}$

 

Chứng minh

 

Ta có $\sqrt{2}$ là một chặn trên của $A$ nên  $Sup\,A \leq \sqrt{2}$.

 

Đặt  $d=\sqrt{2}-Sup\, A\geq 0$, nếu $d>0$ thì theo nguyên lý Archimedes, tồn tại số nguyên dương $n$ sao cho $nd>1$.

 

Đặt  $Sup \,A=x$ và  $m=[nx]+1$ ta có $nx+1>[nx]+1>nx$  hay  $nx+1>m>nx$

 

Suy ra   $x+\frac{1}{n}>\frac{m}{n}>x$  mà  $x+\frac{1}{n}<x+d=Sup\,A+d=\sqrt{2}$

 

Điều này dẫn đến  $\sqrt{2}>\frac{m}{n}>Sup\,A$   (mâu thuẫn)

 

Vậy  $\sqrt{2}-Sup\,A=d=0$   hay   $Sup\,A=\sqrt{2}$

 


Bài toán 2:   Tìm Supremum và Infimum của

 

a)   $A=\left \{ \frac{1}{2n}\,|\,n\in \mathbb{N}^* \right \}$

 

b)   $B=\left \{ \frac{n}{n+2}\,|\,n\in \mathbb{N}^* \right \}$

 

c)   $C=\left \{ 0.2\,,\,0.22\,,\,0.222\,,\,... \right \}$

 

Giải

 

a)   Ta có $\frac{1}{2n}\leq \frac{1}{2}\,\,,\,\,\forall n\geq 1$ nên $\frac{1}{2}$ là một chặn trên của $A$ và $\frac{1}{2}\in A$ nên theo định lý 1 thì $Sup\,A=\frac{1}{2}$

 

Ta thấy dãy $\left ( \frac{1}{2n} \right )_{n\in\mathbb{N}^*}$  là một dãy giảm và  $\lim_{n \to \infty }\frac{1}{2n}=0$  nên theo định lý 4 thì  $Inf \,A=\lim_{n \to \infty }\frac{1}{2n}=0$

 

b)   Ta thấy dãy $\left ( \frac{n}{n+2} \right )_{n\in\mathbb{N}^*}$  là một dãy tăng và  $\lim_{n \to \infty }\frac{n}{n+2}=1$  nên theo định lý 3 thì  $Sup \,B=\lim_{n \to \infty }\frac{n}{n+2}=1$

 

Với mọi $n\geq 2$ thì $\frac{n}{n+2}\geq \frac{1}{2}$ suy ra $\frac{1}{2}$ là một chặn dưới của $B$, mà $\frac{1}{2}\in B$ nên theo định lý 2 thì  $Inf\,B=\frac{1}{2}$

 

c)   Ta thấy  $C=\left \{ \frac{2}{10}\,,\,\frac{2}{10}\left ( 1+\frac{1}{10} \right )\,,\,\frac{2}{10}\left ( 1+\frac{1}{10}+\frac{1}{10^2} \right )\,,\,... \right \}=\left \{ \frac{2}{9}\left ( 1-\frac{1}{10^n}\right )\,|\,n\in \mathbb{N}^* \right \}$

 

Suy ra  $\frac{2}{10}\leq c<\frac{2}{9}$  với mọi $c\in C$, từ đây ta suy ra được  $Inf\,C=\frac{2}{10}$  và  $C$ bị chặn trên.

 

Đặt  $Sup\,C=s$ suy ra $s\leq \frac{2}{9}$ , ta có  $s-\frac{2}{9.10^n}\leq \frac{2}{9}\left ( 1-\frac{1}{10^n} \right )\leq s$   (**)

 

Cho  $n \to \infty$  thì từ (**) ta có  $s\leq \frac{2}{9}\leq s$.

 

Vậy  $Sup\,C=s=\frac{2}{9}$




#731363 $\sum\limits_{k = 2}^n {\frac{1}{{{S_k}}}} > 2\...

Đã gửi bởi phuc_90 on 28-10-2021 - 22:28 trong Bất đẳng thức - Cực trị

Cho $n\epsilon \mathbb{N}, n\geq 2$. Đặt $a_n=1+\frac{1}{2}+...+\frac{1}{n}$ và $S_n=\sum_{i=2}^{n}\frac{a_i}{i}$. Chứng minh rằng với $n> 3$

$\frac{1}{S_2}+\frac{1}{S_3}+...\frac{1}{S_n}> 2(\frac{1}{a_2a_3}+\frac{1}{a_3a_4}+...+\frac{1}{a_{n-1}a_n})$

:D Mong được thảo luận

 

Bổ đề 1:   $a_n > \frac{2n}{n+1}\,\,\,,\,\,\forall n\geq 2$

 

Thật vậy, với $n=2$ thì $a_2=\frac{3}{2}>\frac{4}{3}$, giả sử $a_n > \frac{2n}{n+1}\,\,\,,\,\,n\leq k$.

 

Ta có $a_{k+1}=a_k+\frac{1}{k+1} > \frac{2k}{k+1}+\frac{1}{k+1}=\frac{2k+1}{k+1}>\frac{2(k+1)}{k+2}$

 

Vậy theo nguyên lý quy nạp bổ đề 1 được chứng minh.

 

Bổ đề 2:   $S_n < \frac{a_na_{n-1}}{2}\,\,\,,\,\,\forall n\geq 3$

 

Thật vậy, với $n=3$ thì $S_3=\frac{49}{36} < \frac{33}{24}=\frac{a_2a_3}{2}$

 

Giả sử $S_n < \frac{a_na_{n-1}}{2}\,\,\,,\,\, n\leq k$, khi đó  $S_{k+1}=S_k+\frac{a_{k+1}}{k+1} < \frac{a_ka_{k-1}}{2}+\frac{a_{k+1}}{k+1}$

 

Ta có $\frac{a_{k+1}a_k}{2}-\frac{a_ka_{k-1}}{2}-\frac{a_{k+1}}{k+1}$

 

$=\frac{a_k}{2}\left ( a_{k+1}-a_{k-1} \right )-\frac{a_{k+1}}{k+1}$

 

$=\frac{a_k}{2}\left ( \frac{1}{k}+\frac{1}{k+1} \right )-\frac{a_{k+1}}{k+1}$

 

$=\frac{\left ( 2k+1 \right )a_k-2ka_{k+1}}{2k(k+1)}$

 

$=\frac{\left ( 2k+1 \right )a_k-2k\left ( a_k+\frac{1}{k+1} \right )}{2k(k+1)}$

 

$=\frac{a_k-\frac{2k}{k+1}}{2k(k+1)}$

 

Theo bổ đề 1 thì $\frac{a_{k+1}a_k}{2}-\frac{a_ka_{k-1}}{2}-\frac{a_{k+1}}{k+1}>0$  hay $\frac{a_ka_{k-1}}{2}+\frac{a_{k+1}}{k+1}<\frac{a_{k+1}a_k}{2}$  hay  $S_{k+1}<\frac{a_{k+1}a_k}{2}$

 

Theo nguyên lý quy nạp ta đã chứng minh được bổ đề 2.

 

Trở lại bài toán

 

Theo bổ đề 2, thì   $\frac{1}{S_3}+...+\frac{1}{S_n}>2\left ( \frac{1}{a_2a_3}+...+\frac{1}{a_{n-1}a_n} \right )$

 

Suy ra    $\frac{1}{S_2}+\frac{1}{S_3}+...+\frac{1}{S_n}>2\left ( \frac{1}{a_2a_3}+...+\frac{1}{a_{n-1}a_n} \right )$




#731360 Cho $p\in \mathbb{P}$;$p=3k+2$. CM:...

Đã gửi bởi phuc_90 on 28-10-2021 - 20:54 trong Số học

Cho $p$ là số nguyên tố lẻ có dạng $3k+2$. Chứng minh rằng nếu $a^{2}+ab+b^{2}$ chia hết cho $p$ thì cả $a$ và $b$ đều cùng chia hết cho p biết rằng $a$ và $b$ đều nguyên dương

 

Theo định lý Fermat $\left\{\begin{matrix}a^{p}\equiv a\,\,(mod \,p)\\ b^{p}\equiv b\,\,(mod \,p)\end{matrix}\right.$   $\Rightarrow$   $\left\{\begin{matrix}a^{p+1}\equiv a^2\,\,(mod \,p)\\ b^{p+1}\equiv b^2\,\,(mod \,p)\end{matrix}\right.$

 

Khi đó  $\left ( a^3-b^3 \right )\left ( a^{3k}+a^{3k-3}b^3+...+a^3b^{3k-3}+b^{3k} \right )=a^{3k+3}-b^{3k+3}=a^{p+1}-b^{p+1}\equiv a^2-b^2\,\,(mod \,p)$

 

Ta có $p\,|\left ( a-b \right )\left ( a^2+ab+b^2 \right )=a^3-b^3$    nên    $p \,| a^2-b^2=(a-b)(a+b)$   $\Rightarrow$   $p\,| a-b$ hoặc $p\,| a+b$

 

Trường hợp:  $p\,| a-b$ thì từ $p\,|a^2+ab+b^2=\left ( a-b \right )^2+3ab$   $\Rightarrow$   $p\,| 3ab$  $\Rightarrow$  $p\,| a$ hoặc $p\,| b$

 

Nếu $p\,| a$ thì $p\,| a-(a-b)=b$

 

Nếu $p\,| b$ thì $p\,| a-b+b=a$

 

Trường hợp:  $p\,| a+b$ thì từ $p\,|a^2+ab+b^2=\left ( a+b \right )^2-ab \quad \Rightarrow \quad p\,| ab$  $\Rightarrow$  $p\,| a$ hoặc $p\,| b$

 

Lập luận như trên thì ta luôn có $a\,,\,b$ đều chia hết cho $p$




#731298 Nếu $|A|=k \ne 0$, hãy tính $|2A-3I|$ theo $k...

Đã gửi bởi phuc_90 on 24-10-2021 - 21:36 trong Đại số tuyến tính, Hình học giải tích

Cho A là ma trận vuông cấp 3 thỏa mãn  $A^2-3A+2I=0$

a, Chứng minh: $A$ khả nghịch

b, Tìm $A^{-1}$ theo $A$ và $I$

c, Nếu $|A|=k \ne 0$, hãy tính $|2A-3I|$ theo $k$

 

Ta có  $A^2-3A+2I_n=0 \,\,\,\Rightarrow \,\,\,A\left ( \frac{3}{2}I_n-\frac{1}{2}A \right )=\left ( \frac{3}{2}I_n-\frac{1}{2}A \right )A=I_n$

 

nên $A$  khả nghịch và  $A^{-1}=\left ( \frac{3}{2}I_n-\frac{1}{2}A \right )$

 

Đặt $|2A-3I_n|=a\in \mathbb{R}$ , ta lại có $(2A-3I_n)(2A-3I_n)=I_n$

 

$\Rightarrow$     $a^2=|2A-3I_n||2A-3I_n|=|(2A-3I_n)(2A-3I_n)|=|I_n|=1$

 

Vậy  $|2A-3I_n|=1$  nếu $a>0$  hoặc  $|2A-3I_n|=-1$  nếu  $a<0$




#731202 LÂM ĐỒNG 2022

Đã gửi bởi phuc_90 on 17-10-2021 - 22:59 trong Thi HSG cấp Tỉnh, Thành phố. Olympic 30-4. Đề thi và kiểm tra đội tuyển các cấp.

 

KỲ THI CHỌN HỌC SINH VÀO ĐỘI TUYỀN BỒI DƯỠNG THI HSG QG NĂM 2022
 
Câu 1. (3.0 điểm) Giải phương trình sau trên tập số thực:
$$2x^3-x^2+\sqrt[3]{2x^3-3x+1}=3x+1+\sqrt[3]{x^2+2}$$
 
Câu 2. (4.0 điểm) Đặt $f(n)=(n^2+n+1)^2+1$. Cho $a_n=\frac{f(1).f(3)...f(2n-1)}{f(2).f(4)...f(2n)}$ với $n$ là số nguyên dương. Chứng minh rằng $\lim n\sqrt{a_n}=\frac{1}{\sqrt{2}}$

 

 

Câu 1:   Đặt $u=\sqrt[3]{2x^3-3x+1}$ và $v=\sqrt[3]{x^2+2}$ thì phương trình trên trở thành $u^3-v^3+u-v=0$

 

hay $(u-v)(u^2+uv+v^2+1)=0$ , vì $u^2+uv+v^2+1=(u+\frac{v}{2})^2+\frac{3v^2}{4}+1>0$

 

nên $u=v$ hay $2x^3-3x+1=u^3=v^3=x^2+2 \Rightarrow 2x^3-x^2-3x-1=0 \Rightarrow (2x+1)(x^2-x-1)=0$

 

Phương trình này cho ta 3 nghiệm $x_1=-\frac{1}{2}\,\,,\,\,x_2=\frac{1+\sqrt{5}}{2}\,\,,\,\,x_3=\frac{1-\sqrt{5}}{2}$

 

Câu 2:   Ta có $f(n)=(n^2+n+1)^2+1=(n^2+1)^2+2n(n^2+1)+n^2+1=(n^2+1)((n+1)^2+1)$

 

Khi đó  $a_n=\frac{f(1).f(3)...f(2n-1)}{f(2).f(4)...f(2n)}$

 

$=\frac{(1^2+1)(2^2+1)(3^2+1)...((2n-1)^2+1)((2n)^2+1)}{(2^2+1)(3^2+1)(4^2+1)(5^2+1)...((2n)^2+1)((2n+1)^2+1)}$

 

$=\frac{2}{(2n+1)^2+1}$

 

Suy ra  $\lim_{n \to \infty } n\sqrt{a_n}=\lim_{n \to \infty }\sqrt{\frac{2n^2}{(2n+1)^2+1}}=\frac{1}{\sqrt{2}}$
 




#731193 ĐỀ DỰ TUYỂN MÔN TOÁN NĂM 2021-2022 TRƯỜNG PTNK HCM

Đã gửi bởi phuc_90 on 17-10-2021 - 15:51 trong Thi HSG cấp Tỉnh, Thành phố. Olympic 30-4. Đề thi và kiểm tra đội tuyển các cấp.

             ĐẠI HỌC QUỐC GIA TP.HCM                                            ĐỀ DỰ TUYỂN MÔN TOÁN NĂM 2021-2022

       TRƯỜNG PHỔ THÔNG NĂNG KHIẾU                                                  Ngày thi :   25/09/2021

----------------------------------------------------------                            Thời gian làm bài 180 phút, không kể thời gian phát đề

 

Bài 1. (5,0 điểm)

 

Cho các số thực dương $a,b,c$ thỏa $a^2+b^2+c^2=3$

 

a)   Chứng minh rằng    $a^{4n}+b^{4n}+c^{4n}+a^nb^n+b^nc^n+c^na^n\geq 6 \,\,\,,\,\,\forall n\in \mathbb{N}$

 

b)   Hỏi bất đẳng thức trên còn đúng khi thay $n=\frac{2}{3}$ ?

 

Bài 2. (5,0 điểm)

 

Cho $n$ là số nguyên dương chẵn, có tổng các ước nguyên dương của nó là số lẻ. Chứng minh rằng tổng các ước chính phương (nhỏ hơn $n$) của $n$ sẽ không nhỏ hơn $\frac{n}{4}$

 

Bài 3. (5,0 điểm)

 

Cho tam giác $ABC$, gọi $A_1\,,\,B_1\,,\,C_1$ lần lượt là các điểm đối xứng của $A\,,\,B\,,\,C$ qua $BC\,,\,CA\,,\,AB$

Chứng minh rằng   $A_1\,,\,B_1\,,\,C_1$  thẳng hàng khi và chỉ khi  $cosA\,cosB\,cosC\,\,=\,\,-\frac{3}{8}$

 

Bài 4. (5,0 điểm)

 

Một quốc gia có $99$ thành phố, khoảng cách giữa hai thành phố bất kì không vượt quá $1000$ km. Hai thành phố thuộc quốc gia này được gọi là "xa nhau" nếu khoảng cách giữa chúng lớn hơn hoặc bằng $500\sqrt{2}$ km. Hỏi quốc gia này có bao nhiêu cặp thành phố xa nhau ?

 

                                                                ---------------------------  HẾT ---------------------------------




#731191 Với các số thực a, b, c thỏa mãn $1 \leq a, b, c \leq 2$,...

Đã gửi bởi phuc_90 on 17-10-2021 - 15:22 trong Bất đẳng thức và cực trị

Giả sử $a\leq b\leq c$.

Ta có $\frac{(b-a)(b-c)}{ab}\Rightarrow \frac{b}{a}+\frac{c}{b}\leq 1+\frac{c}{a}$.

Tương tự $\frac{(b-a)(b-c)}{bc}\leq 0\Rightarrow \frac{b}{c}+\frac{a}{b}\leq 1+\frac{a}{c}$.

Do đó $\left ( \frac{1}{a}+\frac{1}{b}+\frac{1}{c} \right )(a+b+c)\leq 5+2\frac{a}{c}+2\frac{c}{a}$.

Suy ra $VT\leq \left(\frac{2a}{c}+\frac{2c}{a}+5\right)\left(1+\frac{1}{a}+\frac{1}{c}\right)=7\left(\frac{1}{a}+\frac{1}{c}\right)+\frac{2a}{c^2}+\frac{2c}{a^2}+\frac{2a}{c}+\frac{2c}{a}+5\leq 7\left(\frac{1}{a}+\frac{1}{c}\right)+\frac{2a}{c^2}+\frac{2c}{a^2}+2(a+c)+5=A+5$.

Ta lại có $\frac{(a-1)(a-2)}{a}\leq 0\Rightarrow a\leq 3-\frac{2}{a}$. Tương tự $c\leq 3-\frac{2}{c}$ nên $A\leq 12+3\left(\frac{a}{c}+\frac{c}{a}\right)+\frac{2c}{a^2}+\frac{2a}{c^2}$.

Ta chứng minh: $3\left ( \frac{a}{c}+\frac{c}{a} \right )+\frac{2c}{a^2}+\frac{2a}{c^2}\leq 3+\frac{3}{c}+\frac{2}{c^2}+2c$. (*)

$(*)\Leftrightarrow 2c\left ( 1-\frac{1}{a^2} \right )+3\left ( 1-\frac{1}{a} \right )\geq \frac{2}{c^2}(a-1)$

$\Leftrightarrow \left ( a-1 \right )\left ( \frac{2c(a+1)}{a^2}+\frac{3}{a}-\frac{2}{c^2} \right )\geq 0$. (luôn đúng do $a-1\geq 0$ và $\frac{2c(a+1)}{a^2}+\frac{3}{a}-\frac{2}{c^2}=2c\left ( \frac{1}{a}+\frac{1}{a^2} \right )+\frac{3}{a}-\frac{2}{c^2}\geq \frac{3c}{2}+\frac{3}{2}-\frac{2}{c^2}>0$).

Suy ra ta chỉ cần chứng minh $3+\frac{3}{c}+\frac{2}{c^2}+2c\leq 10\Leftrightarrow \frac{(c-1)(2c^2-5c-2)}{c^2}\leq 0$. (luôn đúng)

Do đó $A\leq 12+10=22\Rightarrow VT\leq 27$.

Đẳng thức xảy ra khi và chỉ khi $a=b=c=1$.

 

Chỗ chữ màu xanh hình như có vấn đề ?

 

Ta luôn có $\frac{a}{c}+\frac{c}{a}\geq \frac{1}{a}+\frac{1}{c}$




#731145 $\sum_{\sigma \in S_n}sgn(\sigma )\,...

Đã gửi bởi phuc_90 on 14-10-2021 - 17:44 trong Đại số tuyến tính, Hình học giải tích

Bài toán:   Cho $A$ là ma trận vuông cấp $n$ và $\tau \in S_n$

 

Chứng minh rằng         $$\sum_{\sigma \in S_n} sgn(\sigma )\,\,a_{1\sigma (1)}\,\,a_{2\sigma (2)}...\,a_{n\sigma (n)} = \sum_{\sigma \in S_n}sgn(\sigma )\,\,a_{\tau (1)\sigma \tau (1)}\,\,a_{\tau (2)\sigma \tau (2)}...\,a_{\tau (n)\sigma \tau (n)}$$

 

Trong đó, $\tau \sigma = \tau\circ \sigma$




#731102 Tìm tất cả đa thức $P(x),Q(x)$ sao cho $P(Q(x))=Q(P(x))$

Đã gửi bởi phuc_90 on 11-10-2021 - 13:49 trong Đa thức

 

Ta đặt $\displaystyle \deg P( x) =\deg Q( x) =n$ và đặt $\displaystyle R( x) =P( x) -Q( x)$ với $\displaystyle \deg R( x) =k\leqslant n-1$. Ta sẽ chỉ ra $\displaystyle R( x)$ là đa thức đồng nhất hằng. Chú ý rằng ta có thể tách 
$P( P( x)) -Q( Q( x)) =P( P( x)) -Q( P( x)) +Q( P( x)) -Q( Q( x)) =R( P( x)) +Q( P( x)) -Q( Q( x))$
 
Đặt $\displaystyle Q( x) =\sum _{i=1}^{n} a_{i} .x^{i}$ trong đó $\displaystyle a_{n} =1$. 
$Q( P( x)) -Q( Q( x)) =P( x)^{n} -Q( x)^{n} +\sum _{i=1}^{n-1} a_{i}\left[ P( x)^{i} -Q( x)^{i}\right]$
 
Mặt khác $\displaystyle \deg\left( P( x)^{n} -Q( x)^{n}\right) =\deg\left( R( x)\left(\sum _{i=1}^{n-1} P( x)^{i} Q( x)^{n-i-1}\right)\right) =n^{2} -n+k$ nên $\displaystyle \deg( Q( P( x)) -Q( Q( x))) =n^{2} -n+k$ và  $\displaystyle \deg( R( P( x)) +Q( P( x)) -Q( Q( x))) =max\left\{R( P( x)) ,n^{2} -n+k\right\} =n^{2} -n+k$ vì $\displaystyle nk\leqslant n^{2} -n+k$ nên rõ ràng đây là điều không thể xảy ra do vế trái là đa thức 0. Vậy $\displaystyle \deg R( x) =0$ hay $\displaystyle R( x) \equiv c$ và ta có $\displaystyle P( x) =Q( x) +c$ . Thay vào 
 
$Q( P( x)) +c=$$Q( Q( x) +c) +c=Q( Q( x))$
Từ đây đặt $\displaystyle Q( x) =t$ thì suy ra $\displaystyle Q( t+c) +c=Q( t)$ với vô số giá trị $\displaystyle t$ nên $\displaystyle c=0$. Dẫn tới $\displaystyle P( x) \equiv Q( x)$

 

Lời giải này lập luận còn thiếu sót, không rõ ràng, không chính xác.

 

Thiếu sót:  Thiếu trường hợp $deg P \neq deg Q$

 

Không rõ ràng:   Nếu $deg P=deg Q=n$ thì hệ số của biến có số mũ cao nhất của 2 đa thức bạn đang xét tới là chúng bằng 1 hay khác 1. Nếu chúng cùng bằng 1 thì $k\leq n-1$, còn chúng khác nhau thì $k\leq n$.

 

Chỗ lập luận được bôi màu xanh dương. Nếu $k=n-1$ thì sao ? Lúc này $\displaystyle \deg( R( P( x)) +Q( P( x)) -Q( Q( x))) =max\left\{R( P( x)) ,n^{2} -n+k\right\} =n^{2} -n+k=n^2-1$  và  $deg (P(P(x))-Q(Q(x))=n^2-1$ (ở đây tôi xem như bạn đang xét hệ số của biến có số mũ cao nhất của 2 đa thức là bằng 1) thì làm sao có điều vô lý ở đây

 

Không chính xác:   Chỗ lập luận được bôi màu đỏ, bạn phán $VT=P(P(x))-Q(Q(x))$ bằng 0, tôi cũng chào thua :icon6:




#731082 Tìm tất cả đa thức $P(x),Q(x)$ sao cho $P(Q(x))=Q(P(x))$

Đã gửi bởi phuc_90 on 10-10-2021 - 14:49 trong Đa thức

Bài toán:   Tìm tất cả đa thức $P(x)\,,\,Q(x)\in \mathbb{Z}[x]$ sao cho $P(Q(x))=Q(P(x))$




#731081 $2^x=x+1$

Đã gửi bởi phuc_90 on 10-10-2021 - 14:45 trong Phương trình - hệ phương trình - bất phương trình

Giải phương trình sau trên tập số thực: $2^x=x+1$.

 

Đặt $f(x)=2^x-x-1$ , ta thấy $f(0)=f(1)=0$ nên $f(x)$ có nghiệm là   $0\,,\,1$

 

Ta có $f'(x)=2^xln2-1$

 

Cho $f'(x)=0$ ta tìm được nghiệm của $f'(x)$ là   $x_0=-\frac{ln(ln2)}{ln2}\in (0,1)$

 

Bây giờ, nếu $x<0$ thì $f'(x)<0$   suy ra   $f(x)>f(0)=0$   hay  $2^x>x+1$

 

Nếu $0<x\leq x_0$  thì  $f'(x)<0$   suy ra   $f(x)<f(0)=0$   hay  $2^x<x+1$

 

Nếu $x_0<x<1$  thì $f'(x)>0$   suy ra   $f(x)<f(1)=0$   hay  $2^x<x+1$

 

Nếu $1<x$  thì $f'(x)>0$   suy ra   $f(x)>f(1)=0$   hay  $2^x>x+1$

 

Vậy $0\,,\,1$ là tất cả nghiệm của phương trình




#731067 $ P(-x^{2}-x-1)=x^{4}+2x^{3}+2022x^{2...

Đã gửi bởi phuc_90 on 09-10-2021 - 21:07 trong Đa thức

Tìm tất cả các đa thức thỏa mãn với mọi x thuộc R, biết: $ P(-x^{2}-x-1)=x^{4}+2x^{3}+2022x^{2}+2021x+2019 $

 

 

Đặt $G(x)=-x^2-x-1$

 

Từ giả thiết ta có $P(-x^2-x-1)=(x^2+x+1-1)(x^2+x+1+2020)+2019\,\,,\,\,\forall x\in \mathbb{R}$

 

$\Rightarrow P(G(x))=(-G(x)-1)(-G(x)+2020)+2019=(G(x))^2-2019G(x)-1\,\,,\,\,\forall x\in \mathbb{R}$

 

Giả sử $P(x)$ có bậc là $n$ và đa thức $H(x)$ có bậc $n-3$ sao cho $P(x)=x^3H(x)+x^2-2019x-1\,\,,\,\,\forall x\in \mathbb{R}$   (*)

 

Khi đó, $(G(x))^2-2019G(x)-1=P(G(x))=(G(x))^3H(G(x))+(G(x))^2-2019G(x)-1\,\,,\,\,\forall x\in \mathbb{R}$

 

$\Rightarrow (G(x))^3H(G(x))=0\,\,,\,\,\forall x\in \mathbb{R}$

 

Ta thấy $-x^2-x-1=-(x+\frac{1}{2})^2-\frac{3}{4}<0\,\,,\,\,\forall x\in \mathbb{R}$

 

Suy ra đa thức $G(x)=-x^2-x-1$ luôn khác 0 với mọi $x\in \mathbb{R}$

 

Từ đó suy ra $H(G(x))=0\,\,,\,\,\forall x\in \mathbb{R}$, điều này dẫn đến đa thức $H(x)$ có bậc $n-3$ nhưng có vô số nghiệm trong $\mathbb{R}$

 

Suy ra $H(x)=0\,\,,\,\,\forall x\in \mathbb{R}$, từ (*) suy ra được $P(x)=x^2-2019x-1$ chính là đa thức cần tìm




#731062 $e^{A+B}=e^A\,e^B=e^B\,e^A$

Đã gửi bởi phuc_90 on 09-10-2021 - 17:36 trong Đại số tuyến tính, Hình học giải tích

Bài toán:   Cho $A,\,B$ là các ma trận vuông cấp $n$, cùng lũy linh và giao hoán nhau. Đặt  $e^A=\sum_{i=0}^{+\infty }\frac{1}{i!}\,A^i$

 

Chứng minh rằng  $e^{A+B}=e^A\,e^B=e^B\,e^A$

 

Ma trận $A$ được gọi là lũy linh nếu tồn tại $n\in \mathbb{N}$ sao cho $A^n=0$




#731021 Tính tổng Sn (khó)

Đã gửi bởi phuc_90 on 07-10-2021 - 09:25 trong Mệnh đề - tập hợp

Cộng 2 số hạng đầu tiên, rồi lấy kết quả này cộng với số hạng tiếp theo...lặp lại quá trình này sẽ ra kết quả.



#731010 Chứng minh rằng $F(x)={{[P(x)]}^{2}}+1...

Đã gửi bởi phuc_90 on 06-10-2021 - 22:27 trong Đa thức

 

Đa thức $P(x)$ với hệ số nguyên thỏa mãn $\left\{\begin{matrix} P(2006)=2006! \\ xP(x-1)=(x-2006)P(x),\forall x\in \mathbb{R}. \end{matrix}\right.$
 
Chứng minh rằng $F(x)={{[P(x)]}^{2}}+1$ bất khả quy trên $\mathbb{Z}$. 

 

 

Bổ đề 1:   Nếu $H(x)$ là đa thức hệ số nguyên với   $deg H < \infty$   thỏa $\left\{\begin{matrix}H(a_0)=1\,\,,\,\,a_0\in \mathbb{Z}\\H(x)=H(x-1)\,\,,\,\,\forall x\in \mathbb{R} \end{matrix}\right.$       thì      $H(x)=1$

 

Thật vậy, từ điều kiện $\left\{\begin{matrix}H(a_0)=1\,\,,\,\,a_0\in \mathbb{Z}\\H(x)=H(x-1)\,\,,\,\,\forall x\in \mathbb{R} \end{matrix}\right.$

 

Ta suy ra được $H(n)=1\,\,,\,\,\forall n\in \mathbb{Z}$ , khi đó đa thức $H(x)-1$ sẽ có vô số nghiệm trên tập các số nguyên nên      $H(x)-1\equiv 0$

 

Suy ra $H(x)=1$

 

Bổ đề 2:   Đa thức $P(x)=(x-a_1)^2(x-a_2)^2...(x-a_n)^2+1$ là bất khả quy trên $\mathbb{Z}[x]$ với $a_1,a_2,..,a_n$ là các số nguyên

 

Giả sử    $P(x)=Q(x)R(x)$ với $Q(x),R(x)$   là các đa thức hệ số nguyên và    $1\leq deg Q\,\,,\,\,deg R\leq n-1$

 

Ta có    $Q(a_i)R(a_i)=P(a_i)=1\,\,,\,\,i=\overline{1,n}$

 

Suy ra     $Q(a_i)=R(a_i)=1$ hoặc $Q(a_i)=R(a_i)=-1$  với mọi $1\leq i\leq n$

 

Khi đó đa thức    $Q(x)-R(x)$ có $deg(Q-R)\leq n-1$    nhưng có tới $n$ nghiệm là     $a_1,a_2,..,a_n$

 

Do đó    $Q(x)-R(x)\equiv 0$ hay $Q(x)=R(x)$

 

Khi đó ta có    $Q^2(x)=P(x)=(x-a_1)^2(x-a_2)^2...(x-a_n)^2+1$

 

suy ra   $\left ( Q(x)-(x-a_1)...(x-a_n) \right )\left ( Q(x)+(x-a_1)...(x-a_n) \right )=1$ (điều này không thể xảy ra)

 

Vậy $P(x)$ là đa thức bất khả quy trên  $\mathbb{Z}[x]$

 

Trở lại bài toán

 

Ta có $\left\{\begin{matrix}0.P(-1)=-2006.P(0)\\ 1.P(0)=-2005.P(1)\\ ....................\\ 2006.P(2005)=0.P(2006)\end{matrix}\right.$     suy ra    $P(0)=P(1)=...=P(2005)=0$

 

Khi đó ta có thể viết   $P(x)$   dưới dạng    $P(x)=x(x-1)...(x-2005)H(x)$   với   $H(x)$   là đa thức có hệ số nguyên và    $deg H < deg P$

 

Từ điều kiện    $xP(x-1)=(x-2006)P(x)\,\,,\,\,\forall x\in \mathbb{R}$

 

Ta suy ra được   $x(x-1)...(x-2006)H(x-1)=x(x-1)...(x-2006)H(x)\,\,,\,\,\forall x\in \mathbb{R}$

 

Điều này chỉ xảy ra khi     $H(x)=H(x-1)\,\,,\,\,\forall x\in \mathbb{R}$

 

Mặt khác, ta có    $2006!=P(2006)=2006! H(2006)$   suy ra   $H(2006)=1$

 

Khi đó áp dụng các bổ đề trên ta có    $P^2(x)+1$    bất khả quy trên    $\mathbb{Z}[x]$




#730921 Cho $P(x)=Q(x)+Q(1-x)$ và $P(0)=0$. Tính $P(P(2013))...

Đã gửi bởi phuc_90 on 04-10-2021 - 16:48 trong Đa thức

Cho các đa thức $P(x),Q(x)$ với hệ số thực thoả mãn điều kiện $P(x)=Q(x)+Q(1-x),\forall x\in \mathbb R$. Biết $P(0)=0$ và các hệ số của $P(x)$ đều không âm. Tính $P(P(2013))$.

 

Từ giả thiết $P(x)=Q(x)+Q(1-x)$ , ta thay $x$ bởi $1-x$ khi đó $P(1-x)=Q(1-x)+Q(x)$

 

Suy ra  $P(x)=P(1-x)\,\,\,,\,\,\,\forall x\in \mathbb{R}$   (*)

 

Giả sử $P(x)=a_0+a_1x+...+a_nx^n$  với các hệ số là các số thực không âm.

 

Từ (*) cho $x=0$ ta có $a_0=P(0)=P(1)=a_0+a_1+...+a_n$  suy ra  $a_1+a_2+...+a_n=0 \Rightarrow a_1=0,\,\,a_2=0,\,\,...,\,\,a_n=0$

 

Suy ra  $P(x)=a_0$  mà ta lại có $P(0)=0$  nên  $a_0=0$. Vậy $P(x)=0$

 

Điều này dẫn tới $P(P(2013))=0$




#730853 $$2011x+y=3z^2$$

Đã gửi bởi phuc_90 on 02-10-2021 - 16:05 trong Số học

Bài toán:   Tìm tất cả nghiệm nguyên dương $x,y,z$ của phương trình sau

 

$$2011x+y=3z^2$$




#730839 $G=\left \{ x,y\in G\,\,|\,\, x^...

Đã gửi bởi phuc_90 on 01-10-2021 - 16:58 trong Góc Tin học

Bài toán:  Cho nhóm $G$ được xác định như sau  $G=\left \{ x,y\in G\,\,|\,\, x^3=y^2=(xy)^2 =1\right \}$

 

Hãy liệt kê tất cả phần tử, tất cả nhóm con chuẩn tắc của nhóm G.




#730838 chứng minh ma trận vuông $A$ giao hoán với mọi ma trận vuông cùng c...

Đã gửi bởi phuc_90 on 01-10-2021 - 16:36 trong Đại số tuyến tính, Hình học giải tích

Hai ma trận vuông $A$ và $B$ được gọi là giao hoán nhau nếu $AB= BA.$ Chứng minh rằng ma trận vuông $A$ giao hoán với mọi ma trận vuông cùng cấp với nó khi và chỉ khi $A$ có dạng đường chéo $k\cdot l$ với số thực $k,$ và ma trận đơn vị $I.$

 

-  Nếu $A=kI_n$ thì $AB=kI_nB=kB=BkI_n=BA$

 

-  Giả sử $A$ giao hoán với mọi ma trận vuông có cùng cấp với nó

 

Ta lấy $B$ là ma trận đường chéo, có các phần tử khác nhau, thì theo bài https://diendantoanh...t-ma-trận-chéo/  suy ra $A$ là ma trận đường chéo

 

Bây giờ ta cho các phần tử trên đường chéo chính của $A$ bằng nhau thì $A$ sẽ có dạng $kI_n$




#730836 Chứng minh rằng mọi ma trận giao hoán với $A$ cũng là một ma trận c...

Đã gửi bởi phuc_90 on 01-10-2021 - 16:14 trong Đại số tuyến tính, Hình học giải tích



Cho $A$ là một ma trận vuông có các phần tử nằm ngoài đường chéo chính bằng $0,$ gọi là ma trận chéo; với các phần tử trên đường chéo chính khác nhau từng đôi một. Chứng minh rằng mọi ma trận giao hoán với $A$ cũng là một ma trận chéo.

 

Giả sử $A,B$ là ma trận vuông cấp n

 

Do $AB=BA$ nên với mọi $1\leq i\neq j\leq n$ ta có $[AB]_{ij}=[BA]_{ij}$

 

$ \Rightarrow \sum_{i=1}^{n}[A]_{ik}[B]_{kj}=\sum_{i=1}^{n}[B]_{ik}[A]_{kj}\,\,\,\Rightarrow \,\,\, [A]_{ii}[B]_{ij}=[B]_{ij}[A]_{jj}$

 

do $[A]_{ii}\neq [A]_{jj}$  nên $[B]_{ij}=0$

 

Vậy $B$ là ma trận đường chéo.




#730821 $\bigcup_{x\in G}x^{-1}Hx \neq G$

Đã gửi bởi phuc_90 on 30-09-2021 - 21:48 trong Đại số đại cương

Bài toán:   Cho $H$ là một nhóm con thật sự của nhóm $G$ hữu hạn.

 

Chứng minh rằng  $\bigcup_{x\in G}x^{-1}Hx \neq G$